6
$\begingroup$

Suppose $K$ is an algebraic number field, and $a \in K$. Let $n$ be a positive integer. The polynomial $t^n - a \in K[t]$ splits as a product of irreducible factors of degrees $d_1, \dots, d_r$.

Is it necessarily the case that the smallest of the $d_i$ divides all the others?

More generally: is the set $\{d_1, \dots, d_r\}$ always totally ordered by divisibility?

$\endgroup$
4
  • $\begingroup$ Please use a high-level tag like meta.mathoverflow.net/q/1075 $\endgroup$ Commented Dec 2 at 0:51
  • 1
    $\begingroup$ Here is some lazy analysis of question 1 I can't quite finish: Letting $L$ be the splitting field of $x^n - a$ over $K$ and considering the action of $\operatorname{Gal}(L/K)$ on the roots, we have that the $d_i$ are the sizes of the orbits. Thus the question becomes whether there is any subgroup $S$ of $Aff(\mathbb Z / (n)) = \{\begin{pmatrix} a & b \\ 0 & 1 \end{pmatrix} | a \in \mathbb Z / (n) ^\times, b \in \mathbb Z / (n)\}$ such that the size of the smallest orbit of the action of $S$ on $\mathbb Z / (n)$ given by $x \mapsto a x + b$ does not divide the sizes of the larger ones. $\endgroup$ Commented Dec 2 at 19:38
  • 1
    $\begingroup$ My temptation is to analyze this action prime by prime, and indeed when $n$ is an (odd?) prime power I think I can show the statement holds. However I get stuck when trying to glue together my understanding at each prime into some global understanding, the problem is that $S$ might be a good deal smaller than the direct product of its images modulo each prime dividing $n$. $\endgroup$ Commented Dec 2 at 19:45
  • $\begingroup$ I think I have successfully completed a proof along the lines suggested in my earlier comments that the answer to question 1 is yes. I will write it up sometime in the next two weeks if no one else does (remind me if I forget). The point is that if one proves that not only does the statement hold for $n = p^e$, but also that the smallest orbit is a power of $p$ in this case, then the smallest orbit modulo $p_1^{a_1} p_2^{a_2} ... p_k ^ {a _ k}$ is just the product of the smallest orbits modulo each $p_i^{a_i}$, which divides the lcm of any collection of orbits mod the $p_i^{a_i}$ one chooses. $\endgroup$ Commented 21 hours ago

1 Answer 1

8
$\begingroup$

The answer to the second question is negative. For example, if $K=\mathbb{Q}$ and $a=1$, then the $d_i$'s are the $\varphi(m)$'s for $m\mid n$, in some order. In particular, if $n=35$, then $$\{d_1, \dots, d_r\}=\{1,4,6,24\},$$ which is not totally ordered by divisibility.

$\endgroup$
3
  • 1
    $\begingroup$ The smallest degree for which the second question admits a negative answer is $21$. An example is $X^{21}−1$ over the quadratic subfield of the cyclotomic field $\mathbb Q(\zeta_7)$. The degrees will be $1,2 , 3, 3, 6, 6$. $\endgroup$ Commented Dec 2 at 15:02
  • 1
    $\begingroup$ Thanks to you both! This is helpful. $\endgroup$ Commented Dec 2 at 19:18
  • $\begingroup$ @BenWilliams Thank you. Regarding the first question, my guess is that the answer is "yes" for $K=\mathbb{Q}$, but "no" in general. $\endgroup$ Commented Dec 2 at 19:21

You must log in to answer this question.

Start asking to get answers

Find the answer to your question by asking.

Ask question

Explore related questions

See similar questions with these tags.